Momento angular: prueba de valores propios enteros o semienteros

Estoy confundido acerca de una prueba que mi libro de texto de Mecánica Cuántica ha dejado "como ejercicio para el lector".

Entonces, tenemos el operador de momento angular L ^ . También tenemos el momento angular generalizado j ^ : L ^ = j ^ . Tenemos las relaciones de conmutación [ j k ^ , j yo ^ ] y [ j 2 ^ , j k ^ ] .

Hemos introducido los "operadores de escalera" j + ^ = 1 2 ( j 1 ^ + i j 2 ^ ) y j ^ = 1 2 ( j 1 ^ i j 2 ^ ) .

Luego, pasamos a probar tres propiedades para los valores propios y los vectores propios de j 2 ^ y j 3 ^ : j 2 ^ | j , metro = j 2 | j , metro , j 3 ^ | j , metro = metro | j , metro :

  1. metro 2 j 2 (así que hay mínimos y máximos metro s).

  2. j + "sube" metro a metro + 1 , j "baja" metro a metro 1 .

  3. j (que viene de j 2 j ( j + 1 ) ) es un número entero o medio entero.

La pregunta que hace mi libro de texto es: ¿ Por qué Δ metro un numero entero?

Pensé que era por la segunda propiedad, pero cuando le pregunté a mi profesor, me dijo que no era una buena prueba. " j + cambiando metro de 0 a 1 no prueba que Δ metro = 1 / 3 es imposible".

Entonces, ¿cómo demuestro esto? Pensé que era bastante trivial, pero resultó que no lo es.

PD: Ya vi esta pregunta pero no me ayuda mucho.

Editar: es posible que me haya perdido un poco en la traducción. La verdadera pregunta que hace mi libro de texto es ¿ Por qué es Δ metro un numero entero?

Estoy confundido, ¿cómo probaste que j es un número medio entero? Pensé que podría ser un entero o un medio entero (ver mi respuesta aquí: physics.stackexchange.com/q/27899 ). Una vez que haya probado eso para j, estoy de acuerdo con usted en que la conclusión para m se deriva automáticamente de la segunda declaración en su pregunta
Lo siento, quise decir "número entero o medio entero". Estaba considerando los números enteros como compuestos por un número par de medios enteros. Además, quise decir "¿Por qué es Δ metro un número entero?" Lo sé metro puede tener cualquier valor, pero Δ metro (el paso de uno metro al siguiente) solo puede ser un valor entero.

Respuestas (2)

Sus puntos, 1-3 están bien. Hay un valor máximo y mínimo de metro . Llame al valor máximo METRO (tenemos que llamarlo de alguna manera). Ahora podemos aplicar el operador inferior cualquier número de veces, cada vez que reduce el valor de metro por una cantidad entera entera. El valor máximo y mínimo tienen una diferencia finita d . Así que si redondeas d hasta el entero más cercano norte ves que aplicando el operador de descenso norte los tiempos deben producir el estado de menor metro (o de lo contrario golpear primero un estado de magnitud cero). Entonces, un número finito de aplicaciones del operador de reducción envió el valor máximo METRO al valor mínimo, por lo que difieren en una cantidad entera (cada vez que bajó, metro bajó por 1). Entonces los valores máximo y mínimo de metro difieren por un número entero.

Para mí, esta es la prueba de que j = METRO es un valor entero o medio entero ( norte = j ( j ) = 2 j ). Parece que sus pruebas están al revés y también está tratando de probar una afirmación falsa (que metro debe ser entero cuando, por ejemplo, el espín de una partícula de espín 1/2 puede tener metro = 1 / 2 ).

Para mostrar explícitamente que m=1/2 es posible, sea j X = 3 / 4 σ X , j y = 3 / 4 σ y , j z = 3 / 4 σ z y j 2 = 2 3 / 4 ( σ X 2 + σ y 2 + σ z 2 ) . Luego observe que satisfacen las relaciones de conmutación. Luego observe que los valores propios de j z son ± / 2 por eso metro = ± 1 / 2 por definición.

Por lo tanto, es imposible probar su afirmación deseada de que metro es un número entero de la hipótesis ya que el párrafo anterior satisface la hipótesis y, sin embargo, la conclusión es falsa como metro = 1 / 2 no es un número entero, pero es un valor perfectamente fino.

Respuesta a la pregunta editada

Si tienes dos valores de metro que difieren en un número no entero, entonces el operador de reducción aplicado muchas veces a cada uno no puede detenerse en uno y el mismo valor más bajo metro estado. Así que tendría que haber un estado además del más bajo. metro estado que es enviado a cero por el operador de descenso.

Muestre (o asuma) que eso no puede suceder y ya casi ha terminado.

Gracias por tu respuesta, pero cometí un error. Mi libro de texto preguntó por qué es Δ metro un valor entero y creo que respondiste mi pregunta. PD: Conozco el espín medio y su gran importancia en la mecánica cuántica :)

Tu punto 1. muestra que si j (ficticio > 0 ) es el valor máximo de metro , entonces j es el valor más bajo, es decir, las condiciones son simétricas en metro .

Tu punto 2. muestra que debes ser capaz de alcanzar j de j utilizando un número entero de pasos, que es lo mismo que decir 2 j debe ser un entero.

En cuanto a su pregunta final: dado que probó que j ± aumente o disminuya en 1, comience con el valor máximo de metro , cual es j y "bajar" usando j . Solo puedes llegar a estados con metro valores dados por j , j 1 , , j .

Suponga, por el bien de la discusión, que su j = 4 / 3 , de modo que 2 j no es un número entero. Aplicar j produce repetidamente la secuencia de metro valores 4 / 3 , 1 / 3 , 2 / 3 . Es fácil ver que los más pequeños metro no es el negativo del mayor metro ; esta secuencia de metro 's no tiene significado físico ya que invertir el z el eje debe simplemente invertir el signo de la proyección metro , justificando la simetría en el signo de metro encapsulado en su punto 1. Además, nunca obtiene nada más que Δ metro = ± 1 .